1/r 的傅里叶变换

先做 $e^{-ar}/r$ 的傅里叶变换,再令 $a=0$,得 $1/r$ 的傅里叶变换。

$e^{-ar}/r$ 的傅里叶变换为

\begin{equation} \begin{split} \mathcal F[e^{-ar}/r]=&\int d^3\vec{r}\frac{e^{-ar}}{r}e^{-i\vec{q}\cdot\vec{r}}\\ =&\int_0^{\infty} \frac{e^{-ar}}{r}r^2dr\int_0^{\pi}\sin\theta e^{-iqr\cos\theta}d\theta\int_0^{2\pi}d\phi\\ =&2\pi\int_0^{\infty} \frac{e^{-ar}}{r}r^2dr\int_0^{\pi} e^{-iqr\cos\theta}d(-\cos\theta)\\ =&2\pi\int_0^{\infty} \frac{e^{-ar}}{r}\frac{1}{iqr}\left ( e^{iqr\cos\theta}-e^{-iqr\cos\theta} \right )r^2dr\\ =&\frac{4\pi}{q}\int_0^{\infty} e^{-ar}\sin (qr)dr\\ =&\frac{4\pi}{q}\mathrm{Im}\left [\int_0^{\infty} e^{-ar}e^{iqr}dr \right ]\\ =&\frac{4\pi}{q}\mathrm{Im}\left [\int_0^{\infty} e^{(iq-a)r}dr \right ]\\ =&\frac{4\pi}{q}\mathrm{Im}\left [ \frac{e^{(iq-a)r}}{iq-a} \right ]\Bigg |_0^{\infty}\\ =&-\frac{4\pi}{q}\frac{e^{-ar}[q\cos(qr)+a\sin(qr)]}{q^2+a^2}\Bigg|_0^{\infty}\\ =&\frac{4\pi}{q^2+a^2} \end{split} \label{FTa} \end{equation}

令 $a=0$,得 $\frac{1}{r}$ 的傅里叶变换:

\begin{equation}
\mathcal F\left[\frac{1}{r}\right]=\frac{4\pi}{q^2}
\label{FTa0}
\end{equation}

标签: 傅里叶变换

已有 9 条评论

  1. 杜江玮 杜江玮

    老师,您好!我想问一下,第一行中的Fourier变换的基函数exp(-iq·r)为什么最后内积展开后变成了exp(-iqr cosθ)?关键为什么是θ?因为在球坐标下,θ指代跟极轴的夹角,而这里q显然是一个常矢量,但方向不一定指向z轴,通常认为这个内积后cos函数里的角度变量,是q与r的夹角。这个问题不知道该如何解决。谢谢!其他都明白的。

  2. 周睿 周睿

    请问点电荷在平面外的时候如何给这个平面上的势进行傅里叶变换啊?感谢指点

    1. 电荷在原点,平面为$z=d$,做$\frac{1}{\sqrt{d^2+r^2}}$的傅里叶变换。

      1. 周睿 周睿

        请问1/sqrt(d^2+r^2)的傅里叶变换如何求得啊,搞了几天没搞出来,感谢您得指导

        1. 陈婧乐 陈婧乐

          您好,想问下您算出来了吗,我最近也在算这个,卡住了,打扰您,不胜感激

      2. 周睿 周睿

        请问是否有办法从单个点电荷的三维库伦势的傅里叶变换中得出某个平面的库伦势能的傅里叶变换呢?

        1. 感觉不行。不过,我的数学不是太好。

  3. west west

    请问您有二维库伦势的Fourier变换过程吗。。
    二维的积分遇到点问题TAT

    1. 1/q。
      参考这个链接:http://sep.stanford.edu/public/docs/sep103/jon3/paper_html/node3.html

添加新评论

captcha
请输入验证码